Đến nội dung

Stranger411 nội dung

Có 85 mục bởi Stranger411 (Tìm giới hạn từ 24-05-2020)



Sắp theo                Sắp xếp  

#432485 Tìm số tự nhiên $n>4$ nhỏ nhất sa0 ch0 tồn tại $n$ ng...

Đã gửi bởi Stranger411 on 03-07-2013 - 11:14 trong Tổ hợp và rời rạc

Bài toán :

Tìm số tự nhiên $n>4$ nhỏ nhất sa0 ch0 tồn tại $n$ người thỏa mãn : 2 người quen nhau thì không có người quen chung còn 2 người không quen nhau thì có đúng 2 người quen chung !

Trước tiên ta chứng minh những người này có cùng người quen với nhau tại đây.
Cho mỗi người là một đỉnh của đồ thị. 2 người quen nhau thì được nối với nhau bằng 1 cạnh. Nên số người quen của mỗi người sẽ bằng với bậc của đỉnh. Suy ra đồ thì này có các đỉnh cùng bậc. Nói cách khác đây là một đồ thị đẳng cấu. Đồ thị này có $n$ đỉnh, các đỉnh có chung bậc $d$. Suy ra đồ thì có số cạnh là $m= \frac{nd}{2}$ với $d \ge 2$.

Ta tiếp tục chứng minh $|m| \leq 3(|n|-2)$.
Vì đồ thị này là liên thông nên theo định lí Euler suy ra: $|r|=|m|-|n|+2$ (r là số miền)
Gọi $n$ là tổng số cạnh thuộc các miền $r$ thì do mỗi cạnh chỉ thuộc nhiều nhất 2 miền nên $n \leq 2m$. Mỗi miền có ít nhất 3 cạnh nên $n \geq 3r$ từ đó suy ra $r \leq \frac{2}{3}m$. Thay vào công thức Euler ta đc đpcm.

Từ đó, ta có: $m= \frac{nd}{2} \le 3n-6$ $\rightarrow 12 \le (6-d)n$. Vậy $d \le 6$ (ta cần vế phải dương)

Theo các kết quả trên, ta thử chọn các trường hợp và chọn được $n_{min}=8$ với $d=3$.
Vậy $n_{min}=8$.




#433417 Tìm số tự nhiên $n>4$ nhỏ nhất sa0 ch0 tồn tại $n$ ng...

Đã gửi bởi Stranger411 on 07-07-2013 - 00:26 trong Tổ hợp và rời rạc

Anh thử xem lại lời giải xem, người số 2 và 8 đâu thỏa mãn ạ. Đáp số $n=16$

khâu thử chọn anh làm sai @@!
đáp án đúng là $n=16$




#375486 $\frac{n}{m}=\sum_{k=1}^{p-...

Đã gửi bởi Stranger411 on 06-12-2012 - 00:19 trong Số học

Cho $p>3$ là một số nguyên tố.

Đặt $\begin{cases}{n\over m}=\sum_{k=1}^{p-1}{1\over k}\\ \mathrm{gcd}(n,m)=1\end{cases}$

Chứng minh rằng: $p\big| n$

Bài này có nhiều cách giải lắm thầy ạ :D
Theo em cách giải ngắn gọn nhất là sử dụng $\mathbb{Z}/p\mathbb{Z}$

Lời giải:
Ta xét bài toán trong $\mathbb{Z}/p\mathbb{Z}$, do tính tồn tại duy nhất của nghich đảo modulo, ta có:
$\overline{1}^{-1} + \overline{2}^{-1} +...+ \overline{p-1}^{-1} = \overline{1} + \overline{2} +...+ \overline{p-1} = \overline{1+2+...+ p-1} = \overline{\frac{p(p-1)}{2}} \equiv 0$
Q.E.D



#346612 $\sum\limits_{k = 1}^{p - 1} {\l...

Đã gửi bởi Stranger411 on 13-08-2012 - 23:32 trong Số học

Cho p là số nguyên tố .CMR $\sum\limits_{k = 1}^{p - 1} {\left\lfloor {\frac{{{k^3}}}{p}} \right\rfloor } = \frac{{(p + 1)(p - 1)(p - 2)}}{4}$

Khuya rồi !! Lên Vmf chém thử vài bài cho vui :D

Với $1 \leqslant k \leqslant p - 1$thì $\left\{ \begin{gathered}
{k^3} \equiv 0(\bmod p) \\
{\left( {p - k} \right)^3} \equiv - {\left( k \right)^3} \\
\end{gathered} \right.$
Nên $\left( {\frac{{{k^3}}}{p} - \left\lfloor {\frac{{{k^3}}}{p}} \right\rfloor } \right) + \left( {\frac{{{{\left( {p - k} \right)}^3}}}{p} - \left\lfloor {\frac{{{{\left( {p - k} \right)}^3}}}{p}} \right\rfloor } \right) = 1$.
Từ đó, ta được:
$\begin{gathered}
\sum\limits_{k = 1}^{p - 1} {\left\lfloor {\frac{{{k^3}}}{p}} \right\rfloor } = \sum\limits_{k = 1}^{p - 1} {\frac{{{k^3}}}{p}} - \sum\limits_{k = 1}^{p - 1} {\left( {\frac{{{k^3}}}{p} - \left\lfloor {\frac{{{k^3}}}{p}} \right\rfloor } \right)} \\
= \frac{1}{p}\sum\limits_{k = 1}^{p - 1} {{k^3}} - \frac{1}{2}\sum\limits_{k = 1}^{p - 1} {\left( {\frac{{{k^3}}}{p} - \left\lfloor {\frac{{{k^3}}}{p}} \right\rfloor } \right)} + \left( {\frac{{{{\left( {p - k} \right)}^3}}}{p} - \left\lfloor {\frac{{{{\left( {p - k} \right)}^3}}}{p}} \right\rfloor } \right) \\
= \frac{1}{p}{\left( {\frac{{p\left( {p - 1} \right)}}{2}} \right)^2} - \frac{{p - 1}}{2} = \frac{{\left( {p + 1} \right)\left( {p - 1} \right)\left( {p - 2} \right)}}{4} \\
\end{gathered}$

ps Chú Nguyenta98 post trước mình 4 phút.



#313293 Balkan MO 2012

Đã gửi bởi Stranger411 on 29-04-2012 - 14:46 trong Thi HSG Quốc gia và Quốc tế

Balkan MO 2012 - 28 April 2012

Bài 2. Prove that \[ \sum_{cyc}(x+y)\sqrt{(z+x)(z+y)}\geq 4(xy+yz+zx), \] for all positive real numbers $x,y$ and $z$.

Tuy là đề thi quốc gia nhưng mình thấy bài này khá lỏng :D
Cách 1:
Áp dụng bđt Cauchy-Schwarz, ta có:


$\left( x+y \right)\sqrt{\left( z+x \right)\left( z+y \right)}\ge \left( x+y \right)\left( z+\sqrt{xy} \right)$

$=\left( x+y \right)z+\left( x+y \right)\sqrt{xy}\ge \left( x+y \right)z+2xy$
Cách 2: Bằng cách dùng bổ đề của huymit_95, ta viết lại bđt như sau:
$\sum {\frac{1}{{\sqrt {\left( {x + y} \right)\left( {x + z} \right)} }}} \ge \frac{{4\left( {xy + yz + zx} \right)}}{{\left( {x + y} \right)\left( {y + z} \right)\left( {z + x} \right)}}$
Ta có: $\left( {x + y} \right)\left( {y + z} \right)\left( {z + x} \right) \ge \frac{8}{9}\left( {xy + yz + zx} \right)\left( {x + y + z} \right)$
Nên ta cần chứng minh: $\sum {\frac{1}{{\sqrt {\left( {x + y} \right)\left( {x + z} \right)} }}} \ge \frac{9}{{2\left( {x + y + z} \right)}}$
Và bđt này hoàn toàn đúng với bđt Cauchy-Schwarz:
$\sum {\frac{1}{{\sqrt {\left( {x + y} \right)\left( {x + z} \right)} }}} \ge \sum {\frac{2}{{x + y + 2z}}} \ge \frac{9}{{2\left( {x + y + z} \right)}}$



#346412 $(p+1)(p+2)...(2p-1) \equiv (p-1)! (\mod p^3)$

Đã gửi bởi Stranger411 on 13-08-2012 - 11:02 trong Số học

Chứng minh rằng với số nguyên tố $p>3$ thì
$$(p+1)(p+2)...(2p-1) \equiv (p-1)! (\mod p^3)$$

Một số mở rộng cho Wolstenholme’s Theorem.

File gửi kèm

  • File gửi kèm  aa7144.pdf   189.33K   316 Số lần tải



#345850 $\varphi (5^{m}-1)=5^{n}-1$

Đã gửi bởi Stranger411 on 11-08-2012 - 14:38 trong Số học

Ặc, ý em nó là nếu $a_i-1=0$ thì cái $p_i$ có tồn tại đâu mà số chính phương $mod(p)$ nữa? :D

Ko ko, em nhầm hàng rồi :|
mình ko xét cái $\varphi (5^{m}-1)$ đâu
chỉ cần xét ${5^m} - 1 = {2^a}{p_1}^{{a_1}} \ldots {p_k}^{{a_k}}$.
Từ cái này ta chứng minh 5 là số chính phương $mod(p)$ là ok rồi ;)



#368262 $\sum_{k=0}^{\left\lfloor\frac{n...

Đã gửi bởi Stranger411 on 09-11-2012 - 21:48 trong Tổ hợp và rời rạc

Dạo này box tổ hợp và rời rạc của VMF có vẻ trầm lắng!
Để tránh tình trạng này kéo dài, tôi xin khuấy động bằng một bài nho nhỏ

Cho số nguyên $n\ge 3$. Chứng minh đẳng thức:

$\sum_{k=0}^{\left\lfloor\frac{n}{3}\right\rfloor} \binom{n}{3k} = \dfrac{2^n+(-1)^n\left(3\left\lfloor\frac{n}{3}\right\rfloor-3\left\lfloor\frac{n-1}{3}\right\rfloor-1\right)}{3}$

Chào các anh. Bài này dùng hàm sinh kết hợp với RUF tức là Root of Unity Filter đó ạ :D

Định lí: Cho số nguyên dương $n$. Đặt $k=cos \frac{2 \pi}{n}+i sin \frac{2 \pi}{n}$.
Xét đa thức $f(x)=a_n x^n+a_{n-1} x^{n-1}+...+a_1 x+a_0$. Khi đó:
$a_0 + a_{n} + a_{2n} + ... = \frac{1}{n}[f(1)+f(k)+f(k^2)+...+f(k^{n-1})]$

Bài toán: Đặt $k=cos \frac{2 \pi}{3}+i .sin \frac{2 \pi}{3}$.
Áp dụng định lí trên, ta được:
$\sum_{k=0}^{\left\lfloor\frac{n}{3}\right\rfloor} \binom{n}{3k} = \frac{(1+k)^n + (1+ k^2 )^n + (1+1)^n}{3}$
$=\frac{(-1)^n(k^n + k^{2n}) + 2^n}{3}$
từ đó, ta có kết quả bài toán.

Remark: Ngoài ra, ta còn kết quả khác:
Nếu $n$ chia hết cho 3: $\sum_{k=0}^{\infty } \binom{n}{3k} = \frac{2^n + 2(-1)^n}{3}$
Nếu $n$ không chia hết cho 3: $\sum_{k=0}^{\infty } \binom{n}{3k} = \frac{2^n + (-1)^{n+1}}{3} $


ps hxthanh@: thầy cho em hỏi ạ :D hồi rất lâu rồi ấy,thầy có lập 1 topic tên là "Dãy số đầu năm" trong box Đại số THCS. Sau này mod chuyển đi đâu mất :( Thầy cho em cái link để tham khảo được không ạ, Hồi đó em còn non nên chưa đọc được nhiều :D



#345845 $\varphi (5^{m}-1)=5^{n}-1$

Đã gửi bởi Stranger411 on 11-08-2012 - 14:15 trong Số học

He he, thú thực vs anh là em làm y như anh đó, nhưng cả bài của anh và em đều thiếu một cái quan trọng đó là mũ của $p_i$ có thể bằng $0$ anh hiểu ko (hay $a_i-1=0$ đó) nên chưa làm gì được ở đoạn số chính phương $mod(p)$ đâu :D cho nên bài này vẫn cần phải nghĩ thêm


Very beautiful !! A problem in quadratic residue ;)

Bổ đề: Cho các số nguyên $m,n$ và $a>1$. Ta có: $\gcd \left( {{a^m} - 1,{a^n} - 1} \right) = {a^{\gcd \left( {m,n} \right)}} - 1$

Lời giải bài toán:
Vì ${2^3}|{5^m} - 1 \Rightarrow m = 2k + 1 \Rightarrow {p_i}|5.{\left( {{5^k}} \right)^2} - 1$
Vậy 5 là số chính phương (mod $p_i$) nên ${p_i} \equiv - 1(\bmod 5)$
$$ \Rightarrow \left\{ \begin{gathered}
{\left( { - 1} \right)^k} = 1 \\
{\left( { - 2} \right)^{k + 1}} \equiv 1(\bmod 5) \\
\end{gathered} \right.
\Rightarrow \left\{ \begin{gathered}
k \equiv 0(\bmod 2) \\
k \equiv 3(\bmod 4) \\
\end{gathered} \right.$$
Mâu thuẫn,
Vậy $gcd(m,n)>1$.

Thực ra ko phải nghĩ gì cho nhiều em à :P
Từ đoạn này trở xuống mình xét ${5^m} - 1$ nên chả có liên quan gì đến $a_i -1=0$ cả :-j
Với lại $a_i -1=0$ ko quan trọng khi chứng minh $a=2$ ở phần trên :D

Và từ đẳng thức:
\[{5^n} - 1 = \varphi \left( {{5^m} - 1} \right) = {2^{a - 1}}{p_1}^{{a_1} - 1}...{p_k}^{{a_k} - 1}\prod\limits_{i = 1}^k {\left( {{p_i} - 1} \right)} \]
cho ta ${p_i} \not \equiv 1(\bmod 5)$ cũng chẳng liên quan gì đến $a_i -1=0$ em à :P



#345840 $\varphi (5^{m}-1)=5^{n}-1$

Đã gửi bởi Stranger411 on 11-08-2012 - 13:36 trong Số học

giả sử m,n là các số nguyên dương sao cho $\varphi (5^{m}-1)=5^{n}-1$.cmr UCLN của m và n lớn hơn 1

Very beautiful !! A problem in quadratic residue ;)

Bổ đề: Cho các số nguyên $m,n$ và $a>1$. Ta có: $\gcd \left( {{a^m} - 1,{a^n} - 1} \right) = {a^{\gcd \left( {m,n} \right)}} - 1$

Lời giải bài toán:
Giả sử $gcd(m,n)=1$
Ta xét phân tích cơ sở: ${5^m} - 1 = {2^a}{p_1}^{{a_1}} \ldots {p_k}^{{a_k}}$ với ${p_i} > 2$
Vậy nên:
\[{5^n} - 1 = \varphi \left( {{5^m} - 1} \right) = {2^{a - 1}}{p_1}^{{a_1} - 1}...{p_k}^{{a_k} - 1}\prod\limits_{i = 1}^k {\left( {{p_i} - 1} \right)} \]
Vì ${2^a}|{5^n} - 1$. Kết hợp với bổ đề, ta có: $\gcd \left( {{5^m} - {{1,5}^n} - 1} \right) = 5 - 1 = 4$
Vậy nên $a=2$
Vì ${2^3}|{5^m} - 1 \Rightarrow m = 2k + 1 \Rightarrow {p_i}|5.{\left( {{5^k}} \right)^2} - 1$
Vậy 5 là số chính phương (mod $p_i$) nên ${p_i} \equiv - 1(\bmod 5)$
$$ \Rightarrow \left\{ \begin{gathered}
{\left( { - 1} \right)^k} = 1 \\
{\left( { - 2} \right)^{k + 1}} \equiv 1(\bmod 5) \\
\end{gathered} \right.
\Rightarrow \left\{ \begin{gathered}
k \equiv 0(\bmod 2) \\
k \equiv 3(\bmod 4) \\
\end{gathered} \right.$$
Mâu thuẫn,
Vậy $gcd(m,n)>1$.



#422825 Chứng minh tiếp tuyến

Đã gửi bởi Stranger411 on 01-06-2013 - 15:17 trong Hình học

Cho đường tròn $(O)$ và đường thẳng $d$. Gọi $H$ là hình chiếu của $O$ lên $d$. Cho 2 điểm $A,B \in d$ sao cho $HA =HB$. Lấy $M$ bất kì trên $(O)$. $MH,MA,MB$ lần lượt cắt $(O)$ tại $C,D,E$. Gọi $S$ là giao điểm của d và $DE$. Chứng minh $SC$ là tiếp tuyến của đường tròn $(O)$.




#422841 Chứng minh tiếp tuyến

Đã gửi bởi Stranger411 on 01-06-2013 - 16:36 trong Hình học

Bài toán khá hay đó bạn:

Qua $M$ vẽ đường thẳng song song với $d$,Cắt $(O)$ tại $N$,cắt $OH$ tại $I$.

 

Vì $HA=HB$ và $MN//AB$ nên $M(AB,HN)=-1$,Chiếu chùm điều hòa lên $(O)$ ta có tứ giác $DCEN$

 

Kí hiệu mà bạn dùng chỗ nào là thế nào vậy ?
mình xem lại lí thuyết về chùm điều hòa mà không thấy.




#422851 Chứng minh tiếp tuyến

Đã gửi bởi Stranger411 on 01-06-2013 - 17:28 trong Hình học

Qua $M$ vẽ đường thẳng song song với $d$,Cắt $(O)$ tại $N$,cắt $OH$ tại $I$.

 

Vì $HA=HB$ và $MN//AB$ nên $M(AB,HN)=-1$

Theo mình thấy chỗ này nên chứng minh lại bổ đề sau:

 

Bổ đề 1: Cho $a,b,c,d$ là chùm đường thẳng tâm $O$. Đường thẳng $\delta$ không đi qua $O$, cắt $a,b,c,d$ tại $A,B,C,D$. Đường thẳng $\delta'$ không đi qua $O$, cắt $a,b,c$ tại $A',B',C'$. Khi đó $\delta' // d$ thì $(ABCD)=(A'B'C')$.
 

Áp dụng bổ đề trên thì $M(ABHN)= (ABH) = -1$.




#313754 $$\dfrac{b(a+b)}{(c+a)^2}+\dfrac{c(c+b)}{(a+b)^2}+\d...

Đã gửi bởi Stranger411 on 01-05-2012 - 20:13 trong Bất đẳng thức và cực trị

Bài toán 3.
Cho các số thực dương $a, b, c$ sao cho $abc=1$. Chứng minh rằng :
$$\dfrac{1}{(1+a)^2(b+c)}+\dfrac{1}{(1+b)^2(c+a)}+\dfrac{1}{(1+c)^2(a+b)}\le \dfrac{3}{8}$$


Trần Quốc Anh


Hình đã gửi
Trước tiên, ta chứng minh bổ đề:
$$(a+1)(b+c)\ge \frac{(b+1)(c+1)}{\sqrt{bc}}$$
Có thể dùng Cauchy-Schwwarz để chứng minh hoặc biến đổi tương đương ;)

Bất đẳng thức cần chứng minh trở thành:
$$\sum{\frac{1}{{{(a+1)}^{2}}(b+c)}}\le \sum{\frac{bc}{(a+1)(b+1)(zc+1)}}$$

Vậy, ta chỉ cần chứng minh:
$$(a+1)(b+1)(c+1)\ge \frac{8}{3}(\sqrt{ab}+\sqrt{bc}+\sqrt{ca})$$
Và đây là hệ quả của bđt:

$$(x+y)(y+z)(z+x)\ge \frac{8}{9}(x+y+z)(xy+yz+zx)$$
Bất đẳng thức đã được chứng minh. $\blacksquare$


Ps: anh phuc_90 vào góp vui đi ak :P
Bài 1: Giải bằng SOS mất 3 trang giấy :-ss Mọi người ai có cách giải ngắn hơn ko :-ss



#430322 Đề thi vào lớp 10 THPT chuyên Lê Quí Đôn Đà Nẵng 2013-2014 (Hệ số 2)

Đã gửi bởi Stranger411 on 24-06-2013 - 20:42 trong Tài liệu - Đề thi

 

Bài 4. (1,5 điểm)

          Cho một tháp số (gồm 20 ô vuông giống nhau) như hình vẽ. Mỗi ô vuông được ghi một số nguyên dương n với $1\leq n\leq 20$, hai ô vuông bất kỳ không được ghi cùng một số. Ta quy định trong tháp số này 2 ô vuông kề nhau là 2 ô vuông có chung cạnh. Hỏi có thể có cách ghi nào thỏa mãn điều kiện: Chọn 1 ô vuông bất kỳ (khác với các ô vuông được đặt tên a, b, c, d, e, f, g, h như hình vẽ) thì tổng của số được ghi trong ô đó và các số được ghi trong 3 ô vuông kề với nó chia hết cho 4 ?

1001458_10200372004477893_499076836_n.jp




#375898 cho n là số nguyên dương lớn hơn 1. CMR $2^n-1$ không chia hết cho n

Đã gửi bởi Stranger411 on 07-12-2012 - 22:20 trong Số học

cho n là số nguyên dương lớn hơn 1. CMR $2^n-1$ không chia hết cho n

Ý tưởng ko khác gì mấy vs nguyenta98, chủ yếu là xét $v_2$ thôi.
Giải:
Xét phân tích tiêu chuẩn của $n=\prod\limits_{i = 1}^h {{p_i}^{{k_i}}}$ với các số nguyên tố ${p_1} < {p_2} < ... < {p_h}$. Trong đó ${p_i} = 1 + {2^{{r_i}}}{m_i}$ ($m_i$ lẻ) $\Rightarrow n \equiv 1(\bmod {m})$
Đặt $n - 1 = {2^m}t$ $\Rightarrow {2^{{2^m}t}} \equiv - 1(\bmod {p_i})$
Mà $- 1 \equiv {2^{{2^m}t{m_i}}} \equiv {2^{({p_i} - 1)t}} \equiv 1(\bmod {p_i})$
Vậy $p_i = 2$ (Vô lí)
Q.E.D

Remark: Chắc nguyenta98 có biết bài này:
Tìm tất cả số tự nhiên $n$ sao cho: $n|{2^n} + 2$



#339875 $\sum\limits_{k=1}^{p-1}{({...

Đã gửi bởi Stranger411 on 25-07-2012 - 07:42 trong Tổ hợp và rời rạc

Cho số nguyên tố $p>3$ và tập hợp $M=\left\{ 1,2,...,p \right\}$. Với mỗi số nguyên $k$ thỏa mãn $1\le k\le p$ ta đặt : ${{E}_{k}}=\left\{ A\subset M:|A|=k \right\}$ và ${{x}_{k}}=\sum\limits_{A\in {{E}_{k}}}{\left( \min A+\max A \right)}$. Chứng minh rằng:
$$\sum\limits_{k=1}^{p-1}{({{x}_{k}}C_{p}^{k})\equiv0(\bmod \,\,{{p}^{3}})}$$



#339956 $\sum\limits_{k=1}^{p-1}{({...

Đã gửi bởi Stranger411 on 25-07-2012 - 10:29 trong Tổ hợp và rời rạc

Ở đầu bài toán có đk là p>3
mình sẽ tiếp tục lời giải của bạn để chứng minh chia hết cho $p^3$
ta có $\sum_{k=1}^{p-1}\binom{p}{k}^2=\binom{2p}{p}-2$
mà theo định lí Wolstenholme ta có $\binom{2p}{p} \equiv 2 (mod p^3)$
phát biểu Định lí http://chuyentoanpbc...2/06/trang1.jpg

Em năm nay 12 mà chả biết mấy cái này :mellow:
Em ko bit đánh giá thế nào nên phải dựa vào cách chứng minh của định lí Willson nên nó hơi dài 1 tí :mellow:

Ta chứng minh:
$\sum\limits_{k=1}^{p-1}{{{\left( C_{p}^{k} \right)}^{2}}\equiv 1(\bmod \,\,{{p}^{3}})}$ (1)

$\Leftrightarrow \sum\limits_{k=1}^{p-1}{{{\left( \frac{(p-1)!}{k!(p-k)!} \right)}^{2}}\equiv 0(\bmod \,\,\,p)}$ (2)

Với mỗi $k\in \text{ }\!\!\{\!\!\text{ 1}\text{,2},...,\text{ p-1}\}$ đặt ${{a}_{k}}=\frac{(p-1)!}{k!(p-k)!}$
$ \Leftrightarrow k!.{{a}_{k}}=(p-1)(p-2)...(p-k+1) $
$ \Leftrightarrow k.{{a}_{k}}\equiv {{(-1)}^{k-1}}(\bmod \,\,\,p) $ (3)

Xét ${{b}_{k}}=\frac{(p-1)!}{k}$, $\forall k\in \left\{ 1,2,...,p-1 \right\}$.

Theo Định lý Wison ta có $k{{b}_{k}}\equiv (-1)(\bmod \,\,\,p)$. (4)

Từ (3) và (4) ta có :
${{a}_{k}}\equiv {{(-1)}^{k}}{{b}_{k}}(\bmod \,\,p)$ (5)

Do $p$ là số nguyên tố và $k\in \left\{ 1,2,...,p-1 \right\}$ nên tồn tại duy nhất $j\in \left\{ 1,2,...,p-1 \right\}$ sao cho:
$(kj)\equiv 1(\bmod \,\,\,p)$$\Rightarrow $${{(kj)}^{2}}\equiv 1(\bmod \,\,\,p)$.

Khi đó:
$$\sum\limits_{k=1}^{p-1}{{{({{b}_{k}})}^{2}}}=\sum\limits_{k=1}^{p-1}{\left( {{({{b}_{k}})}^{2}}.1 \right)}\equiv \sum\limits_{k=1}^{p-1}{\left( {{({{b}_{k}})}^{2}}.{{(kj)}^{2}} \right)}\equiv \left( (p-1)! \right)\sum\limits_{j=1}^{p-1}{{{j}^{2}}(\bmod \,\,\,p)}$$

$$\sum\limits_{j=1}^{p-1}{{{j}^{2}}=\frac{p(p-1)(2p-1)}{6}\equiv 0(\bmod \,\,\,p)}$$
nên $\sum\limits_{k=1}^{p-1}{{{({{b}_{k}})}^{2}}}\equiv 0(\bmod \,\,\,p)$ (6)

Từ (5) và (6) suy ra $\sum\limits_{k=1}^{p-1}{{{({{a}_{k}})}^{2}}}\equiv 0(\bmod \,\,\,p)$ hay (2) đúng.



#339925 $\sum\limits_{k=1}^{p-1}{({...

Đã gửi bởi Stranger411 on 25-07-2012 - 09:53 trong Tổ hợp và rời rạc

Có vẻ như bài này chỉ chứng minh chia hết cho $p^2$ thôi, thử với $p=3$ có vẻ không đúng.

Rõ ràng là :
$$ \sum\limits_{k=1}^{p-1}(C_p^{k})^2 \vdots p^2$$

Anh gì đó ơi, bài này có thể quy về chứng minh:
\[\sum\limits_{k=1}^{p-1}{{{\left( C_{p}^{k} \right)}^{2}}\equiv 1(\bmod \,\,{{p}^{3}})}\]
$$\Leftrightarrow \sum\limits_{k=1}^{p-1}{{{\left( \frac{(p-1)!}{k!(p-k)!} \right)}^{2}}\equiv 0(\bmod \,\,\,p)}$$


Với mỗi $k\in \text{ }\!\!\{\!\!\text{ 1}\text{,2},...,\text{ p-1}\}$ đặt ${{a}_{k}}=\frac{(p-1)!}{k!(p-k)!}$
$$ \Leftrightarrow k!.{{a}_{k}}=(p-1)(p-2)...(p-k+1) $$
$$ \Leftrightarrow k.{{a}_{k}}\equiv {{(-1)}^{k-1}}(\bmod \,\,\,p) $$

Đến đây, dùng định lí Willson thôi anh ạ :lol:
Mà $p>3$ mà anh :icon6:



#398293 Những người phát cuồng vì tramyvodoi

Đã gửi bởi Stranger411 on 19-02-2013 - 19:01 trong Góc giao lưu

Post cái hình lên :v
Bạn nào thích thì cứ lấy làm ava lun nhá :-j

Hình đã gửi



#397889 Những người phát cuồng vì tramyvodoi

Đã gửi bởi Stranger411 on 17-02-2013 - 23:58 trong Góc giao lưu

Các cháu trên faceboook đang phát cuồng :D
Mong bạn tramyvodoi đừng giận nhá :))
Hình đã gửi

Và đây là hậu quả của những cháu đua đòi :-ss
Hình đã gửi



#346020 Tìm các số nguyên dương $a,b,c$ sao cho $\frac{a^{2}+b^{2...

Đã gửi bởi Stranger411 on 11-08-2012 - 23:47 trong Số học

He he áp dụng cái bổ đề anh Tường nói thì bài này làm ngon
Giải như sau:
Bổ đề: $p \in \mathbb{P}, p \equiv 2 \pmod{3}, a^2+3b^2 \vdots p \Leftrightarrow p|a,b$

Em Tạ giải kinh quá :P
Chắc thằng Tường nó chả bao giờ theo kịp đâu :P

Cách khác:
Đẳng thức được viết lại như sau $(a+b+c)^2=(3k+2)(ab+bc+ca)$
Chọn số nguyên tố $p$ sao cho $\left\{ \begin{gathered}
{p^{2a - 1}}|3k + 2 \\
{p^{2a}}|3k + 2 \\
\end{gathered} \right.$
$ \Rightarrow \left\{ \begin{gathered}
{p^a}|a + b + c \\
p|ab + bc + ca \\
\end{gathered} \right.$
$c \equiv - a - b(\bmod p) \Rightarrow p|a^2 + ab + b^2 \Rightarrow p|{(2a + b)^2} + 3{b^2}$
$ \Rightarrow \left( {\frac{{ - 3}}{p}} \right) = 1$. Và điều này vô lí vì $p \equiv 2(\bmod 3)$.
Vậy không tồn tại $a,b,c$ thỏa mãn bài toán. $\blacksquare$



#347148 Tìm các số nguyên dương $a,b,c$ sao cho $\frac{a^{2}+b^{2...

Đã gửi bởi Stranger411 on 16-08-2012 - 10:37 trong Số học

sai từ chỗ này và nguyên nhân là do làm tắt $p|{(2a + b)^2} + 3{b^2}$
$ \Rightarrow \left( {\frac{{ - 3}}{p}} \right) = 1$
muốn dùng lengdre(hay tiếng việ gọi là thặng dư toàn phương) trước tiên ta phải đưa nó về dạng (mà ở đây) là
a2$\equiv$-3 (mod p) cái đã,mà ở đây muốn đưa về dạng này ta phải giả sử a không chia hết cho p,''vậy nên thiếu TH a,b chia hết cho p'',mà TH này luôn đúng,nếu không thấy dc thì cho a=b=p ta có 12p2 chia hết cho p ,vì vậy có giải kiểu gì đi nữa vẫn phải thông qua a,b,c chia hết cho p rồi mới giải tiếp,nên không có cách bạn stranger nói

Nói chuyện vs Uyenha cực kì bực mình @@!
Mình và mọi người đã ko muốn nói rồi mà bạn cứ thích cãi cùn.

Trước đó, MOD đã gộp bớt vài bài của bạn để tránh spam trong topic.
Thắc mắc thì ko phải là tội nhưng cứ nói dai như thế người ta chả thích tí nào đâu bạn :)

Mời bạn tham khảo thêm về kí hiệu Lengdre:
File gửi kèm  Cong Thuc Legendre.pdf   67.83K   1709 Số lần tải



#313126 Bài 1. cho $a,b,c>0$có $abc=1$ chứng minh rằng $...

Đã gửi bởi Stranger411 on 28-04-2012 - 14:02 trong Bất đẳng thức - Cực trị

Bài 1. cho $a,b,c>0$có $abc=1$ chứng minh rằng
$\frac{1}{a^3+b+c}+\frac{1}{b^3+a+c}+\frac{1}{c^3+a+b}\leq \frac{3}{a+b+c}$
Bài 2.cho $a,b,c>0 abc=1$ tìm GTNN của
$P=\frac{1}{2a^3+b^3+c^3+2}+\frac{1}{a^3+2b^3+c^3+2}+\frac{1}{a^3+b^3+2c^3+2}$
Bài 3.chứng minh:
$\frac{a^3}{a^2+ab+2b^2}+\frac{b^3}{b^2+bc+2c^2}+\frac{c^3}{c^2+ab+2a^2}\geq \frac{a+b+c}{4}$
Bài 4. $ x+y+z=3$,tìm giá trị nhỏ nhất của biểu thức

$P=\frac{x}{y^3+16}+\frac{y}{z^3+16}+\frac{z}{x^3+16}$

nếu ko có trả lời một tuần nữa minh sẽ post dáp án

Có lẽ cũng chẳng cần post đáp án nữa đâu bạn :lol:
Bài 2:
Áp dụng bđt Cauchy, ta đc:
$$ \frac{1}{2{{a}^{3}}+{{b}^{3}}+{{c}^{3}}+2}\le \frac{1}{4}\left( \frac{1}{{{a}^{3}}+{{b}^{3}}+1}+\frac{1}{{{a}^{3}}+{{c}^{3}}+1} \right) $$
Từ đó, ta được: $ P\le \frac{1}{2}\left( \frac{1}{{{a}^{3}}+{{b}^{3}}+1}+\frac{1}{{{a}^{3}}+{{c}^{3}}+1}+\frac{1}{{{c}^{3}}+{{b}^{3}}+1} \right)\le \frac{1}{2} $

Vì $ {{a}^{3}}+{{b}^{3}}+1\ge ab\left( a+b \right)+1=ab\left( a+b+c \right) $
Bài 3: Sử dụng pp tiếp tuyến, ta chứng minh:

$$ \frac{{{a}^{3}}}{{{a}^{2}}+ab+2{{b}^{2}}}\ge \frac{9a-5b}{16} $$
Cộng các bđt tương tự, ta có đpcm.



#346392 Tìm các số nguyên dương $a,b,c$ sao cho $\frac{a^{2}+b^{2...

Đã gửi bởi Stranger411 on 13-08-2012 - 09:49 trong Số học

muốn có -3 là thặng dư toàn phương của p thì 1 trong 2 số 2a+b hoặc b không chia hết cho p,nên theo nguyenta a,b,c chia hết cho p là hợp lí

Gì nữa đây bạn :P
$\left( {\frac{{ - 3}}{p}} \right) = 1$ là kí hiệu Lengdre ;) Chớ có phải thăng dư toàn phuơng gì đâu ;))
Cách em Nguyenta98 là lùi vô hạn. Cách mình là dùng các định lí về thăng dư bậc 2 thôi ;)